Repeated root in field of char 0

  • Thread starter fishturtle1
  • Start date
  • Tags
    Field Root
In summary: Going through the proof, I realized there was a small mistake. I'll restate it below with the correction:Proof: We want to show if ##\gcd(p(x), p'(x)) \neq 1## then ##p(x)## has a repeated root. We will show the contrapositive. Suppose ##p(x)## has no repeated roots. By Kroneker's theorem, we can find a field ##E## such that ##E## contains ##F## and ##p## splits over ##E##. So, we can write ##p(x) = \prod_{i=1}^{n} (x-a_i)## where ##a_i \in E## and ##a_i \ne
  • #1
fishturtle1
394
82
Homework Statement
If F is a field of char 0 and ##p(x) \in F[x]## is irreducible, then ##p(x)## has no repeated roots. Hint: Consider ##\gcd(p(x), p'(x))##
Relevant Equations
Definition: Let ##F## be a field. A nonzero polynomial ##p(x) \in F[x]## is irreducible over ##F## if ## \deg p(x) \ge 1## and there is no factorization ##p(x) = f(x)g(x)## in ##F[x]## with ##\deg f(x) < \deg p(x)## and ##\deg g(x) < \deg p(x)##.

Definition: Let ##R## be a domain, and let ##f(x), g(x) \in R[x]##. The greatest common divisor of ##f(x)## and ##g(x)## is a polynomial ##d(x) \in R[x]## such that
\\

i) ##d(x)## is a common divisor of ##f(x)## and ##g(x)##;
ii) if ##c(x)## is any common divisor of ##f(x)## and ##g(x)## then ##c(x) \vert d(x)##;
iii) ##d(x)## is monic.

Definition: The prime field of a field ##F## is the intersection of all the subfields of ##F##.

Definition: A field has characteristic ##0## if its prime field is isomorphic to ##\mathbb{Q}##; it has characteristic ##p## if its prime field is isomorphic to ##\mathbb{Z}_p##.

Definition: If ##p(x) = a_nx^n + a_{n-1}x^{n-1} + \dots + a_1x + a_0## then ##p'(x) = na_nx^{n-1} + (n-1)a_{n-1}a^{n-2} + \dots + a_1##.
Proof: We will first show ##\gcd(p(x), p'(x)) = 1##. Define ##d(x) = \gcd(p(x), p'(x))##. Then we can find ##q(x) \in F[x]## such that ##p(x) = d(x)q(x)##. But ##p(x)## is irreducible which means ##d(x)## is constant or ##q(x)## is constant. If ##q(x)## is constant, then ##\deg d(x) = \deg p(x)##. In particular, ##\deg d(x) = \deg p(x) > \deg p'(x)##. But we have also assumed ##d(x) \vert p'(x)##. This implies ##\deg d(x) \le \deg p'(x)##. We have reached a contradiction. So, we can conclude ##d(x)## is constant. Since ##d(x)## is also monic, we have ##d(x) = 1##.

Next, we suppose ##p(x)## has a repeated root, say ##a \in F##. Then we can find ##k(x) \in F[x]## such that ##p(x) = (x-a)^2k(x)##. Taking the derivative, we have ##p'(x) = 2(x-a)k(x) + (x-a)^2k'(x)##. Hence, ##(x-a) \vert \gcd(p(x), p'(x))##. This contradicts the fact ##\gcd(p(x), p'(x)) = 1##.

We can conclude ##p(x)## has no repeated roots. []My question is, I don't think I used the assumption that ##F## has characteristic ##0##. Is there a mistake in the proof?
 
Physics news on Phys.org
  • #2
fishturtle1 said:
Homework Statement:: If F is a field of char 0 and ##p(x) \in F[x]## is irreducible, then ##p(x)## has no repeated roots. Hint: Consider ##\gcd(p(x), p'(x))##
Relevant Equations:: Definition: Let ##F## be a field. A nonzero polynomial ##p(x) \in F[x]## is irreducible over ##F## if ## \deg p(x) \ge 1## and there is no factorization ##p(x) = f(x)g(x)## in ##F[x]## with ##\deg f(x) < \deg p(x)## and ##\deg g(x) < \deg p(x)##.

Definition: Let ##R## be a domain, and let ##f(x), g(x) \in R[x]##. The greatest common divisor of ##f(x)## and ##g(x)## is a polynomial ##d(x) \in R[x]## such that
\\

i) ##d(x)## is a common divisor of ##f(x)## and ##g(x)##;
ii) if ##c(x)## is any common divisor of ##f(x)## and ##g(x)## then ##c(x) \vert d(x)##;
iii) ##d(x)## is monic.

Definition: The prime field of a field ##F## is the intersection of all the subfields of ##F##.

Definition: A field has characteristic ##0## if its prime field is isomorphic to ##\mathbb{Q}##; it has characteristic ##p## if its prime field is isomorphic to ##\mathbb{Z}_p##.

Definition: If ##p(x) = a_nx^n + a_{n-1}x^{n-1} + \dots + a_1x + a_0## then ##p'(x) = na_nx^{n-1} + (n-1)a_{n-1}a^{n-2} + \dots + a_1##.

Proof: We will first show ##\gcd(p(x), p'(x)) = 1##. Define ##d(x) = \gcd(p(x), p'(x))##. Then we can find ##q(x) \in F[x]## such that ##p(x) = d(x)q(x)##. But ##p(x)## is irreducible which means ##d(x)## is constant or ##q(x)## is constant. If ##q(x)## is constant, then ##\deg d(x) = \deg p(x)##. In particular, ##\deg d(x) = \deg p(x) > \deg p'(x)##. But we have also assumed ##d(x) \vert p'(x)##. This implies ##\deg d(x) \le \deg p'(x)##. We have reached a contradiction. So, we can conclude ##d(x)## is constant. Since ##d(x)## is also monic, we have ##d(x) = 1##.
Isn't this automatically the case? If ##p(x)## is irreducible, then ##\operatorname{gcd}(p(x),f(x))=1## for any polynomial ##f(x)##, simply because ##\operatorname{gcd}\,|\,p.##
fishturtle1 said:
Next, we suppose ##p(x)## has a repeated root, say ##a \in F##. Then we can find ##k(x) \in F[x]## such that ##p(x) = (x-a)^2k(x)##. Taking the derivative, we have ##p'(x) = 2(x-a)k(x) + (x-a)^2k'(x)##. Hence, ##(x-a) \vert \gcd(p(x), p'(x))##. This contradicts the fact ##\gcd(p(x), p'(x)) = 1##.

We can conclude ##p(x)## has no repeated roots. []My question is, I don't think I used the assumption that ##F## has characteristic ##0##. Is there a mistake in the proof?
Only a notational. The way you wrote it, you have proven that ##p(x)## is irreducible if and only if ##\operatorname{gcd}(p(x),p'(x))=1.## But we want another conclusion: If ##p(x)## is irreducible with a repeated root ##a##, then ##(x-a)\,|\,p(x) \wedge (x-a)\,|\,p'(x)## so it cannot be irreducible.

But even this can be shortened. If ##p(x)## has a root ##a##, then ##(x-a)\,|\,p(x)## and ##p(x)## is reducible.

It was valuable to prove what you have proven, but it wasn't necessary for your statement. All you need is the Euclidean algorithm to conclude that ##x-a## divides the polynomial if ##a## is a root. But every polynomial ring over a field (of any characteristic) is a Euclidean ring.
 
Last edited:
  • Informative
Likes fishturtle1
  • #3
That makes sense; I see now there was a simpler proof.

Proof: Assume by contradiction ##p(x)## has a repeated root, say ##a \in F##. Since ##F[x]## is a Euclidean ring, we have ##p(x) = (x-a)^2 q(x)## for some ##q(x) \in F[x]##. We can rewrite this as ##p(x) = (x-a) h(x)## where ##h(x) = (x-a)q(x)##. But ##\deg (x-a) < \deg p(x)## and ##\deg h(x) < \deg p(x)##. This shows ##p(x)## is not irreducible. We have reached a contradiction. []

Thank you!
 
  • #4
You should try to prove the following statement:

##p(x)## has a repeated root ##a## if and only if ##(x-a)\,|\,\operatorname{gcd}(p(x),p'(x)).##

You have already shown "##\Longrightarrow ##". Can you show the other direction?
 
  • Like
Likes fishturtle1
  • #5
fresh_42 said:
You should try to prove the following statement:

##p(x)## has a repeated root ##a## if and only if ##(x-a)\,|\,\operatorname{gcd}(p(x),p'(x)).##

You have already shown "##\Longrightarrow ##". Can you show the other direction?
Sure, i'll give it a try:

Proof: We want to show if ##\gcd(p(x), p'(x)) \neq 1## then ##p(x)## has a repeated root. We will show the contrapositive. Suppose ##p(x)## has no repeated roots. By Kroneker's theorem, we can find a field ##E## such that ##E## contains ##F## and ##p## splits over ##E##. So, we can write ##p(x) = \prod_{i=1}^{n} (x-a_i)## where ##a_i \in E## and ##a_i \neq a_j## whenever ##i \neq j##.

Assume by contradiction ##\gcd(p(x), p'(x)) \neq 1##. Then ##(x-a_i) \vert \gcd(p(x), p'(x))## for some ##a_i##. We can find ##q(x)## such that ##p(x) = (x-a_i)q(x)##. Taking the derivative gives

$$p'(x) = 1\cdot q(x) + (x-a_i)q'(x)$$

From the above, we see ##(x-a_i)## does not divide ##p'(x)##. We have reached a contradiction. We can conclude ##\gcd(p(x), p'(x)) = 1##, which proves the contrapositive. []
 
  • Like
Likes fresh_42
  • #6
fishturtle1 said:
Sure, i'll give it a try:

Proof: We want to show if ##\gcd(p(x), p'(x)) \neq 1## then ##p(x)## has a repeated root. We will show the contrapositive. Suppose ##p(x)## has no repeated roots. By Kroneker's theorem, we can find a field ##E## such that ##E## contains ##F## and ##p## splits over ##E##. So, we can write ##p(x) = \prod_{i=1}^{n} (x-a_i)## where ##a_i \in E## and ##a_i \neq a_j## whenever ##i \neq j##.

Assume by contradiction ##\gcd(p(x), p'(x)) \neq 1##. Then ##(x-a_i) \vert \gcd(p(x), p'(x))## for some ##a_i##. We can find ##q(x)## such that ##p(x) = (x-a_i)q(x)##. Taking the derivative gives

$$p'(x) = 1\cdot q(x) + (x-a_i)q'(x)$$

From the above, we see ##(x-a_i)## does not divide ##p'(x)##. We have reached a contradiction. We can conclude ##\gcd(p(x), p'(x)) = 1##, which proves the contrapositive. []
Makes a bit dizzy, doesn't it?

If ##(x-a) \,|\,p(x) \wedge (x-a)\,|\,p'(x)## then ##p(x)=... ## and ##p'(x)=... ## ...
 
  • Like
Likes fishturtle1
  • #7
Thanks! Based off the above:

Proof: Suppose ##(x-a) \vert \gcd(p(x), p'(x))##. Then we can find ##q(x)\in F[x]## such that ##p(x) = (x-a)q(x)##. Taking the derivative of ##p(x)## gives

$$p'(x) = q(x) + (x-a)q'(x)$$

which implies ##(x-a) \vert q(x)##. So, we can find ##k(x) \in F[x]## such that

$$p(x) = (x-a)q(x) = (x-a)(x-a)k(x) = (x-a)^2k(x)$$

i.e., ##p(x)## has a repeated root. []
 
  • Like
Likes fresh_42
  • #8
I think I'm missing something from this thread. Is the goal to demonstrate that p(x) doesn't have a repeated root in F, or is the goal to demonstrate that it doesn't have a repeated root in any field extension of F? Because if the root has to be in F, you get an even simpler result, which is that it has no roots unless it's a linear polynomial right. And it seems like that's what's being assumed here, so I assume we're answering the wrong question.

I'm particular, why is ##q(x)\in F[x]##? If there's a good reason for it I don't see it in the proof.
 
Last edited:
  • #9
Office_Shredder said:
I think I'm missing something from this thread. Is the goal to demonstrate that p(x) doesn't have a repeated root in F, or is the goal to demonstrate that it doesn't have a repeated root in any field extension of F? Because if the root has to be in F, you get an even simpler result, which is that it has no roots unless it's a linear polynomial right. And it seems like that's what's being assumed here, so I assume we're answering the wrong question.

I'm particular, why is ##q(x)\in F[x]##? If there's a good reason for it I don't see it in the proof.
In a previous exercise, the textbook says the following "...Show ##f(x)## has no repeated roots (i.e., ##f(x)## is not a multiple of ##(x-a)^2## for any ##a \in F##.) " So, I think (?) we can assume the repeated root belongs to ##F##.

And then we have, for ##a \in F##, we have ##f(a) = 0 \iff f(x) = (x-a)q(x)## for some ##q(x) \in F[x]##.
 
  • #10
So f is irreducible, and you just factored it into ##k(x)q(x)## where ##k(x)=x-a##. What can you say about ##q(x)##, since f is irreducible?

In particular, the hint seems totally unnecessary for this case.
 
  • Like
Likes fishturtle1
  • #11
In that case, ##q(x)## must be constant. But that means ##f(x)## has degree ##1## which implies ##f(x)## has no repeated roots.

Now that you point it out, it seems (?) likely that the textbook must have meant ##f(x)## has no repeated roots in any field extension. I am not sure.
 
  • #12
##\operatorname{gcd}(f(x),f'(x))## is used to count multiple roots, so it is in that realm. This is e.g. necessary if you consider sign changes in the values ##f(x)## between roots: ##(x-1)^2## has no change of signs from left to right at ##x=1## whereas ##x-1## does have a change of signs. There are theorems which use this fact.
 
  • Informative
Likes fishturtle1
  • #13
Office_Shredder said:
So f is irreducible, and you just factored it into ##k(x)q(x)## where ##k(x)=x-a##. What can you say about ##q(x)##, since f is irreducible?

In particular, the hint seems totally unnecessary for this case.
I am sooo sorry, I just reread the section and the definition of repeated root was updated:

Definition 1: If ##F## is a field and ##f(x) \in F[x]##, then ##f(x)## has repeated roots if there is a field ##E## containing ##F## and a factorization in ##E[x]## of the form $$f(x) = (x-a)^2h(x)$$

-------
I will try again to prove the original problem with the new definition. In the textbook,

Corollary 18: Let ##k \subset K## be fields, and let ##f(x), g(x) \in k[x] \subset K[x]##. Then the ##\gcd## of ##f## and ##g## computed in ##K[x]## is the same as the ##\gcd## of ##f## and ##g## computed in ##k[x]##.

Exercise 44 (from a couple sections back): Let ##f(x) = \prod_{i=1}^{n}(x-a_i) \in F[x]##, where ##F## is a field and ##a_i \in F## for all ##i##. Show that ##f(x)## has no repeated roots (i.e., ##f(x)## is not a multiple of ##(x-a)^2## for any ##a \in F##) if and only if ##\gcd(f(x), f'(x)) = 1##, where ##f'(x)## is the derivative of ##f(x)##.We will use the above two statements to prove the OP problem: If ##F## is a field of characteristic ##0## and ##p(x) \in F[x]## is irreducible, then ##p(x)## has no repeated roots. (Hint: Consider ##\gcd(p(x), p'(x))##.

Proof: Since ##p(x)## is irreducible over ##F##, it's only monic divisors in ##F[x]## are ##1##. So, ##\gcd(p(x), p'(x)) = 1## in ##F[x]##. By Corollary 18, ##\gcd(p(x), p'(x)) = 1## in any field extension ##E## of ##F##. By Exercise 44, this is equivalent to saying ##p(x)## has no repeated roots over any field extension ##E## of ##F##. We can conclude ##p(x)## has no repeated roots (Definition 1). []
 

1. What is a repeated root in a field of characteristic 0?

A repeated root in a field of characteristic 0 is an element that, when multiplied by itself multiple times, results in the additive identity element (usually denoted as 0). In other words, it is an element that has a multiplicative order of 1.

2. How is a repeated root different from a simple root?

A simple root is an element that, when multiplied by itself, results in the identity element. This means that a simple root has a multiplicative order of 2. On the other hand, a repeated root has a multiplicative order of 1, meaning it is only equal to itself when multiplied by itself.

3. Can a repeated root exist in any field of characteristic 0?

No, a repeated root can only exist in fields of characteristic 0. In fields of positive characteristic, such as fields of prime order, every element has a finite multiplicative order and therefore cannot be a repeated root.

4. How is a repeated root related to the concept of algebraic closure?

A field of characteristic 0 that contains a repeated root is not algebraically closed, meaning it does not contain all the roots of all its polynomials. This is because the repeated root cannot be a root of any polynomial with degree higher than 1.

5. Can a repeated root be a part of a field extension?

Yes, a repeated root can be a part of a field extension as long as the extended field is also of characteristic 0. However, the repeated root will still have a multiplicative order of 1 in the extended field.

Similar threads

  • Calculus and Beyond Homework Help
Replies
1
Views
1K
  • Calculus and Beyond Homework Help
Replies
24
Views
803
  • Calculus and Beyond Homework Help
Replies
5
Views
622
  • Calculus and Beyond Homework Help
Replies
7
Views
290
  • Calculus and Beyond Homework Help
Replies
7
Views
558
  • Calculus and Beyond Homework Help
Replies
1
Views
295
  • Calculus and Beyond Homework Help
Replies
13
Views
2K
  • Calculus and Beyond Homework Help
Replies
18
Views
1K
  • Calculus and Beyond Homework Help
Replies
11
Views
3K
  • Calculus and Beyond Homework Help
Replies
1
Views
1K
Back
Top